Kinematics Belt and Pulley Problem

In summary: T=Jα, where T and J are given and T is the max torque. That leaves α as T/J = 26,666.67 and if I differentiate it, as it's a constant, will equal zero.Conversely, if i integrate r1Θ1 = x then I'm left with Θ = 1/2αt2 + 0. This can't be the answer either as the velocity vs time relationship should ascend and the descend because it starts and stops at 0, unless it immediately stops the instant it reaches the desired position (which would theoretically 'minimize' the amount of time required). So I understand the general relationship between (angular) velocity, time and acceleration, but I can
  • #36
whitejac said:
I simply don't know.
It's completely analogous to linear motion. If an object accelerates at constant rate a, how far will it go from rest in time t? Use SUVAT or just integrate.
 
Physics news on Phys.org
  • #37
haruspex said:
It's completely analogous to linear motion. If an object accelerates at constant rate a, how far will it go from rest in time t? Use SUVAT or just integrate.

Capture.PNG

I've never heard of SUVAT, but this is what you mean? Integrating theta twice would yield the second equation regardless.
haruspex said:
If the load accelerates from rest at αL, how long will it take to turn through angle θ/2?

Okay, a load accelerating from rest at αL would have the position equation:
θL = θ0L0Lt+1/2αLt2
when θLL/2 and I.C.'s are applied:
θL = αLt2
And t = SQRT[θLL]
 
  • #38
whitejac said:
View attachment 195980
I've never heard of SUVAT, but this is what you mean? Integrating theta twice would yield the second equation regardless.Okay, a load accelerating from rest at αL would have the position equation:
θL = θ0L0Lt+1/2αLt2
when θLL/2 and I.C.'s are applied:
θL = αLt2
And t = SQRT[θLL]
Right. So minimising time means maximising αL, which is not surprising but worth checking.

In your post #23, you eliminated αL and kept αM. But αL is what we need to maximise. So you need to revisit that and obtain the equation that relates αL to τM, r, R, JL and JM.
 
  • #39
haruspex said:
Right. So minimising time means maximising αL, which is not surprising but worth checking.

In your post #23, you eliminated αL and kept αM. But αL is what we need to maximise. So you need to revisit that and obtain the equation that relates αL to τM, r, R, JL and JM.

Okay, so I have done most of this but am out of town and away from the computer unail Sunday night.

I have rearranged Tm into an expression that has αL. Considering what you said about Radius R being the thing that changes the angular acceleration here, I took the derivative with respect to R and got an expression. To maximise alpha, I set the derivative equal to 0 and then solve for R. This became a heft fraction I cannot easily type on my phone.
Regardless, once I solve this, I will have the maximum apha and thus the Radius that minimizes the time required to go about a theta?
 
  • #40
whitejac said:
Okay, so I have done most of this but am out of town and away from the computer unail Sunday night.

I have rearranged Tm into an expression that has αL. Considering what you said about Radius R being the thing that changes the angular acceleration here, I took the derivative with respect to R and got an expression. To maximise alpha, I set the derivative equal to 0 and then solve for R. This became a heft fraction I cannot easily type on my phone.
Regardless, once I solve this, I will have the maximum apha and thus the Radius that minimizes the time required to go about a theta?
Yes, that should work.
 
  • Like
Likes whitejac
  • #41
haruspex said:
Yes, that should work.
I've finally figured it out! I have to say, I am embarrassed how much I forgot of my multivariable calculator class. I thought I understood it better than this, but I suppose application is the real challenge.
Thank you so much for your patience. I will post back Sunday night with my attempt at part 2 and 3 but I believe it will not be too much of a challenge. It ought be similar in concept to this. The SUVAT has an expression with t, so I would relate it and the new found expression? Forgive me if this doesn't seem coherent. I'm not near paper to validate myself, but wanted to get the main idea straightened so I could mull it over during my trip.
 

Similar threads

  • Introductory Physics Homework Help
Replies
5
Views
491
  • Engineering and Comp Sci Homework Help
Replies
1
Views
1K
  • Introductory Physics Homework Help
3
Replies
78
Views
7K
  • Introductory Physics Homework Help
Replies
4
Views
11K
Replies
2
Views
3K
  • Introductory Physics Homework Help
Replies
9
Views
11K
  • Introductory Physics Homework Help
Replies
7
Views
1K
  • Introductory Physics Homework Help
Replies
1
Views
1K
  • Other Physics Topics
Replies
1
Views
1K
  • Introductory Physics Homework Help
Replies
1
Views
3K
Back
Top